MBE wrong 3 Flashcards

1
Q

Which statement concerning impeachment by evidence of bias is true?
response - incorrect

A A witness’s bias may be shown only by cross-examination, not by extrinsic evidence.

B The evidence of bias must be substantively admissible in the case (not just offered for impeachment purposes).

Correct
C A party is not permitted to show that a witness’s bias is justified.

Incorrect
D A party may introduce extrinsic evidence of a witness’s bias prior to the witness’s testimony.

A

Although a party is permitted to show a witness’s bias or interest, another party may not subsequently show that the witness’s bias is justified. A witness may always be impeached by extrinsic evidence of bias or interest, provided a proper foundation is laid. Even evidence that is substantively inadmissible may be admitted for impeachment purposes if relevant to show bias or interest. Extrinsic evidence of a witness’s bias prior to the witness’s testimony would not be allowed because of foundational requirements. The party must ask the witness about the facts that show bias or interest on cross-examination. If the witness admits those facts, the court then decides whether to allow extrinsic evidence of bias.

How well did you know this?
1
Not at all
2
3
4
5
Perfectly
2
Q

Prior statements by a witness that are consistent with the witness’s testimony at trial are sometimes used to rehabilitate a witness who has been impeached. A party may rehabilitate his witness by introducing a prior consistent statement EXCEPT when:
response - incorrect

A the witness has been impeached by evidence of her prior inconsistent statement on the same subject

Correct
B the witness has been impeached with evidence of a prior criminal conviction

C the witness’s sensory deficiencies have been impeached

Incorrect
D the witness has been impeached by a charge that the witness is lying because of some motive, and the prior consistent statement was made before that motive existed

A

A prior consistent statement cannot be used to rehabilitate a witness whose general character for truthfulness has been impeached, such as by prior criminal convictions or acts of misconduct. On the other hand, prior consistent statements are admissible when the opposing counsel has impeached the credibility of a witness by making an express or an implied charge that the witness is lying or exaggerating because of some motive (e.g., bias), if the prior consistent statement was made by the witness before the time of the alleged motive to lie or exaggerate. Also, when opposing counsel has impeached the credibility of a witness on some non-character ground, such as an alleged inconsistency or sensory deficiency, counsel may introduce a prior consistent statement if, under the circumstances, it has a special tendency to rehabilitate the witness’s credibility.

How well did you know this?
1
Not at all
2
3
4
5
Perfectly
3
Q

A victim and his former business partner, the defendant, had a bitter falling out after the victim accused the defendant of embezzling company funds. The defendant threatened to get even. Shortly thereafter, while driving on the expressway, a car swerved suddenly in front of the victim’s car. Although the victim applied the brakes immediately, his car failed to stop. To avoid colliding with the car ahead of him, he swerved to the right and smashed into a concrete retaining wall. A passing motorist stopped and came to the aid of the victim. Bleeding profusely from a head wound, and rapidly losing consciousness, the victim said, “I don’t think I’m going to make it. I tried to slow down, but my brakes didn’t work. My former partner must have tampered with them to get back at me.” With that, the victim lapsed into unconsciousness, and has been in a coma and on life support ever since. A personal injury suit has been filed on his behalf by a court-appointed guardian against the defendant.

At trial, can the motorist testify as to the statement made by the victim?

A No, because the victim did not know that the defendant tampered with the brakes.

B No, because the victim is still alive.

C Yes, because the victim thought he was about to die.

D Yes, because this is a civil case.

A

Testimony as to the statement made by the victim is inadmissible as a statement under belief of impending death, because the victim did not actually have firsthand knowledge that the defendant was responsible for the collision. The statement is hearsay because it is a statement made by the declarant (the victim), other than while testifying, offered to prove the truth of the matter asserted therein. Here, the plaintiff wants to present this testimony to prove the truth of the statement that the defendant was responsible for the brake failure, and will argue that the statement falls under the hearsay exception for dying declarations. In a civil case or a homicide prosecution, a statement made by a now unavailable declarant while believing his death to be imminent, that concerns the cause or circumstances of what he believed to be his impending death, is admissible. [Fed. R. Evid. 804(b)(2)] For this exception to apply, the declarant need not actually die. Rather, the declarant must be “unavailable” when the statement is offered. A declarant is unavailable if he: (i) is exempted from testifying on the ground of privilege, (ii) refuses to testify despite a court order, (iii) testifies to lack of memory of the subject matter of the statement, (iv) cannot be present or testify because of death or physical or mental illness, or (v) is beyond the reach of the court’s subpoena and the statement’s proponent has been unable to procure his attendance or testimony by process or other reasonable means. Regarding the statement at issue here, the victim certainly thought he was about to die from his injuries. In addition, he is unavailable, as his physical condition prevents him from testifying. However, the victim’s statement represents a mere suspicion that the defendant tampered with the brakes. As well-founded as such a suspicion may be (given the history between the victim and the defendant), a statement based on mere suspicion rather than actual knowledge does not constitute a statement concerning the cause or circumstances of an “impending death” for purposes of the dying declarations exception.

How well did you know this?
1
Not at all
2
3
4
5
Perfectly
4
Q

A wife is on trial for the murder of her husband. She is accused of pushing him from the window of their 13th floor apartment; she claims he committed suicide. The wife called an operator for a suicide-prevention clinic to testify that the deceased husband had called the clinic on more than one occasion, each time telling the operator that he wanted to “end it all.”

Is the testimony admissible?

A

Yes, under state of mind

How well did you know this?
1
Not at all
2
3
4
5
Perfectly
5
Q

A plaintiff sued a chimney sweeping company for personal injury and property damages resulting from an explosion in her chimney the evening after the company had cleaned it. The explosion, which occurred when the plaintiff lit a fire in the fireplace, caused minor damage to the chimney, roof, and to the plaintiff, who was hit by falling bricks. As evidence that she assumed the risk of injury, the company offers to have its foreman testify that he had told the plaintiff not to use the fireplace for 24 hours to allow certain chemicals to evaporate.

Is the foreman’s proposed testimony hearsay?

A No, because the declarant is testifying as a witness at the hearing.

B No, because the statement is not offered for its truth.

C Yes, but it should be admitted under the hearsay exception for present sense impressions.

D Yes, but it should be admitted under the present state of mind exception to the hearsay rule.

A

The evidence is not hearsay because the statement is not offered for its truth; the statement is offered to show its effect on the plaintiff. Hearsay is a statement, other than one made by the declarant while testifying at the trial or hearing, offered in evidence to prove the truth of the matter asserted. [Fed. R. Evid. 801(c)] If a statement is hearsay, and no exception to the hearsay rule is applicable, the evidence must be excluded upon appropriate objection to its admission. [Fed. R. Evid. 802] A statement that would be inadmissible hearsay to prove the truth of the statement may be admitted to show the statement’s effect on the listener or reader. Thus, in a negligence case, where knowledge of a danger is at issue, a statement of warning is admissible for the limited purpose of showing knowledge or notice on the part of a listener. Here, the defense of assumption of the risk has been raised. Whether the plaintiff knew of the danger involved in lighting a fire within 24 hours of the chimney cleaning is an issue. Consequently, the statement of the foreman is admissible to show that the plaintiff had knowledge of the possible danger. The statement is not hearsay because it is not offered to prove that it was in fact dangerous for the plaintiff to light a fire. (A) incorrectly states that the reason the statement is not hearsay is that the declarant is testifying as a witness. The fact that the declarant is now testifying does not alter the hearsay nature of a statement. Any out-of-court statement offered for its truth is hearsay in most jurisdictions (the Federal Rules have a few specific statements characterized as nonhearsay) regardless of whether the declarant is testifying. The reason hearsay is excluded is that there is no opportunity for cross-examination at the time the statement was made. The key in this case is not that the declarant is testifying, but that the statement is not being offered for its truth. (C) characterizes the testimony as hearsay, which is incorrect because it is not being offered for its truth. Even if this testimony were hearsay, it is incorrect to state that it falls under the hearsay exception for present sense impressions, which are statements made contemporaneously with perceiving some event. The testimony of the foreman would not come within this exception.

How well did you know this?
1
Not at all
2
3
4
5
Perfectly
6
Q

The owner of a small business was injured in a traffic accident. A month after the accident, the owner asked an employee to take a photograph of the intersection where the accident occurred. The employee took the photograph and gave it to the owner, who in turn gave it to his lawyer. The lawyer wishes to introduce the photograph into evidence at trial of the owner’s lawsuit against the defendant. The lawyer plans to have the employee testify that he took the photograph. The lawyer also plans to call a witness who lives in the neighborhood of the accident scene and arrived at the intersection shortly after the accident occurred. The witness is willing to testify that the scene in the photograph is in fact the intersection where the accident happened.

Whose testimony is necessary to introduce the photograph into evidence?

response - incorrect

A The employee’s testimony is necessary and the witness’s is unnecessary.

Correct
B The witness’s testimony is necessary and the employee’s is unnecessary.

Incorrect
C The testimonies of both the employee and the witness are necessary.

D The picture is inadmissible.

A

nly the witness’s testimony is necessary to introduce the photograph. To be admissible, a photograph must be identified by a witness as a portrayal of certain facts relevant to the issue, and verified by the witness as a correct representation of those facts. It is sufficient if the witness who identifies the photograph is familiar with the scene or object depicted. It is not necessary to call the photographer to authenticate the photograph. Here, the actual physical appearance of the intersection is most likely relevant to the manner in which the accident occurred. As a resident of the neighborhood in which the accident took place, and as someone who was at the scene of the accident shortly after its occurrence, the witness is sufficiently familiar with the scene to testify that the photograph is an accurate representation of the accident scene. Such identification by the witness is needed for the photograph to be admissible.

(C) is incorrect because, as stated above, the testimony of the employee, the photographer, is not necessary. (D) is incorrect because the photograph is admissible if properly identified by the witness.

How well did you know this?
1
Not at all
2
3
4
5
Perfectly
7
Q

A landowner validly conveyed a small office building to the Green Party “as long as they use it for operating quarters until the next presidential election.” After the next presidential election, which was in three years, the building would go to a private organization that monitors and prepares comprehensive listings of gas prices throughout the country. A year after the conveyance, the landowner died, validly devising all of her property to her son. Although this jurisdiction is a common law jurisdiction with respect to all real property considerations, the state’s probate laws provide that future interests or estates in real property may be passed by will or descent in the same manner as present or possessory interests. Last week, the Green Party and the gas monitoring organization joined together to sell the office building in fee simple absolute to a developer. The son filed suit to prevent the sale of the property to the developer.

In this action, who should prevail?

A The Green Party and the gas monitoring organization, because together they own a fee simple absolute in the building.

B The Green Party and the gas monitoring organization, because the attempted restrictions on the use of the property violate the Rule Against Perpetuities.

C The Green Party and the gas monitoring organization, because the deed restriction was an unlawful restraint on alienation.

D The son, because he did not sign the contract of sale.

A

The son may enjoin the sale because he has an interest in the property. A fee simple determinable is an estate that automatically terminates on the happening of a stated event. The Green Party’s interest in the office building is a fee simple determinable because it lasts as long as the Party is using the building for operating quarters. However, the grant does not provide for the contingency of the Green Party ceasing to use the building as operating quarters before the next presidential election. This gap would be filled by a possibility of reverter retained by the landowner. Because the landowner passed that interest to her son in her will, there can be no contract to sell the property without his signature. Note: Although the gas monitoring organization appears to have an indefeasibly vested remainder (i.e., it is created in an ascertained company, is certain to become possessory, and is not subject to being defeated, divested, or diminished in size), its interest is not capable of taking on the natural termination of the preceding estate and so is characterized as a springing executory interest. (A) is wrong because the son also has an interest in the land

How well did you know this?
1
Not at all
2
3
4
5
Perfectly
8
Q

Several members of a small terrorist group are on trial in federal court for conspiring to bomb a military installation. The prosecution would like to introduce the testimony of a military guard at one of the installation’s gates. The guard had been present when a bomb that was being planted by a member of the group had exploded prematurely. The guard will testify that she ran over to administer first aid to the member, who in great pain told her that his group was in the process of planting three other bombs in other areas of the military installation and was going to detonate them all at the same time to get publicity for their cause. The guard will also testify that the member disclosed the locations of the other bombs and the names of two other members of the group. The authorities were able to prevent the other bombings and arrest the other members of the group. The member died from his injuries.

What is the best basis for allowing the guard to testify as to the member’s statements?

A As a vicarious admission of a co-conspirator.

B As a statement against interest.

C As a statement of present state of mind.

D As a dying declaration.

A

The member’s statements are admissible as a statement against interest. Under the Federal Rules, statements of a person, now unavailable as a witness, against that person’s pecuniary, proprietary, or penal interest when made are admissible as an exception to the hearsay rule. Here, the member’s statements implicating himself in the bombing conspiracy were against his penal interest when he made them; hence, they are probably admissible under that exception. (A) is wrong because for a statement to qualify under the Federal Rules as a vicarious admission of another member of the conspiracy, the admission must have been in furtherance of the conspiracy by a participant in it. Here, the member’s statements were not made in furtherance of the conspiracy but instead served to thwart its success.

How well did you know this?
1
Not at all
2
3
4
5
Perfectly
9
Q

A plaintiff sued a defendant for damages suffered when a load of bricks fell off the defendant’s truck directly in front of the plaintiff while she was driving on a highway. The plaintiff charged that the defendant was negligent in supplying his truck with a defective load chain clamp, which helped tie the load to the bed of the truck, and in failing to secure the load properly on the truck. The plaintiff calls a witness who testifies that he was formerly employed as a truck driver and is an acquaintance of the defendant. The witness further testifies that immediately prior to the accident he had coffee with the defendant at a cafe, and mentioned to the defendant that the tie chains holding the load of bricks looked kind of loose.

Assuming proper objection by the defendant’s attorney, how should the court rule on the admissibility of such testimony?

A Admissible under an exception to the hearsay rule.

B Admissible nonhearsay.

C Inadmissible hearsay.

D Inadmissible opinion evidence.

A

The witness’s testimony is admissible nonhearsay. The statement by the witness is not being offered to prove the truth of the matter asserted therein and thus is not hearsay. Hearsay is a statement made out of court by the declarant, offered in evidence to prove the truth of the matter asserted. [Fed. R. Evid. 801(c)] Although hearsay is inadmissible (unless an exception to the hearsay rule is applicable), a statement that would be inadmissible hearsay to prove the truth thereof may be admitted to show the statement’s effect on the listener or reader. Thus, in a negligence case, where knowledge of a danger is at issue, a person’s warning statement is admissible for the limited purpose of showing knowledge or notice on the part of a listener. Here, one of the theories of recovery underlying the plaintiff’s lawsuit is that the defendant negligently failed to secure the load. Therefore, the plaintiff must show that the defendant either knew or should have known that the load was not properly secured. Consequently, the witness’s statement that the chains looked loose is admissible to show that the defendant had notice of the possible danger. If this same out-of-court statement were offered to show that its contents were true (i.e., that the chains were in fact loose), then it would constitute hearsay, but because the statement is offered to show notice to the defendant of a possible danger, it is nonhearsa

How well did you know this?
1
Not at all
2
3
4
5
Perfectly
10
Q

A defendant is on trial for manslaughter after he hit a victim in a bar, causing the victim to fall and hit his head on the marble bar top. The defendant claims that he hit the victim in self-defense after the victim lunged at him with a knife. During the prosecution’s case, a witness testifies that she heard the victim’s friend shout at the defendant, “You just killed a helpless man!” A defense witness is called to testify that he was there and does not remember hearing the victim’s friend say anything.

Should the defense witness’s testimony be admitted?

A No, it is irrelevant to any issue in the case.

B Yes, it is proper impeachment of the prosecution’s witness.

C No, it is improper impeachment of the prosecution’s witness because it relates to a collateral matter.

D No, it is improper impeachment because it does not positively controvert the prosecution witness’s testimony, as the defense witness merely says he does not remember.

A

The defense witness’s testimony should be admitted as proper impeachment of the prosecution’s witness. Impeachment is the casting of an adverse reflection on the veracity of a witness. A witness may be impeached by either cross-examination or extrinsic evidence, such as by putting other witnesses on the stand who contradict the witness’s testimony. Here, the defense is using the testimony of its witness to impeach the prosecution witness’s testimony as to what the victim’s friend said. This is proper. (A) is wrong because a witness’s credibility is always relevant. Furthermore, the defense witness’s testimony relates to a crucial issue in the case; i.e., whether the victim was armed with a knife or was “helpless.” Thus, the testimony is relevant.

How well did you know this?
1
Not at all
2
3
4
5
Perfectly
11
Q

Do you need physical harm in negligent infliction of emotional distress?

A

Yes and no. is incorrect because physical impact is not required for this tort; the threat of impact is enough. Therefore, even in the absence of impact, the owner could recover if she had been within the zone of danger from the tenant’s negligence.

How well did you know this?
1
Not at all
2
3
4
5
Perfectly
12
Q

A beneficiary has filed a petition in the probate court to contest the validity of a testator’s will. The beneficiary contends that when the testator executed the will eight years before, he had a severe mental illness and was incapable of forming a valid testamentary intent. In support of this contention, the beneficiary seeks to offer an affidavit prepared by the testator’s former attorney, which states that she was asked to prepare a will for the testator just four months before this will was made. The attorney had refused to do so because it was her opinion that the testator seemed incoherent and paranoid. is there attorney client privilege?

A

NO, the observations of the attorney would not be deemed a “communication received from the client.” Also, while the attorney-client privilege generally survives the client’s death, it does not apply to communications relevant to an issue between parties who are claiming through the same deceased client, such as in the probate proceedings here.

How well did you know this?
1
Not at all
2
3
4
5
Perfectly
13
Q

A seeks compensatory and NOT punitive damages. A asks B in discovery for net worth. OK?

A

NO, not worth not relevant when seeking compensatory only.

How well did you know this?
1
Not at all
2
3
4
5
Perfectly
14
Q

A salvage company offered for sale Confederate dollars that had been recovered when the company recently raised a shipwreck off the coast of South Carolina. A purchasing agent for a private west coast museum purchased the bills, but he had represented that he was buying them for himself in hopes of obtaining a lower price.

After purchasing the bills, the agent carefully packaged them and had them shipped to his museum. While the bills were in transit, the museum burned to the ground and its owner decided that she would not rebuild because most of her collections had been destroyed.

When the bills arrived after the fire, the owner opened the package only to discover that the bills were too brittle for shipping by this method—three bills had disintegrated in transit. Undaunted, the owner took the remaining nine bills and had them mounted behind a glass frame so she could display them in her study. While the bills were being framed, the owner read on the Internet that a large cache of similar bills had just been discovered, and the market price for such bills had just been cut in half.

Frustrated but still undaunted, the owner hung the framed bills in her study. Unfortunately, the salt water had reacted with the pigments in the bills in such a way that shortly after they had been exposed to indirect sunlight, all of the color in the bills faded almost completely away. No other Confederate bills raised from the ocean before had similar reactions; these bills appear to have been printed using substandard dyes.

Which of the following facts would give the museum owner the best basis for rescinding the contract with the salvage company?

A The bills were too brittle for transport.

B The discovery of a large cache of similar bills a few days after the sale.

C The bills’ unusual reaction to indirect sunlight.

D The destruction of the museum before the bills arrived.

A

The circumstances of (C) offer the best grounds for rescinding the contract based on mutual mistake. When both parties entering into a contract are mistaken about existing facts relating to the agreement, the contract may be voidable by the adversely affected party if (i) the mistake concerns a basic assumption on which the contract is made; (ii) the mistake has a material effect on the agreed-upon exchange; and (iii) the party seeking avoidance did not assume the risk of the mistake. Here, both parties probably believed that the bills would be suitable for display, like other bills that had been raised from the ocean. They had no reason to suspect that the bills would discolor when exposed to indirect sunlight. This occurrence probably rendered the bills nearly worthless, creating a material imbalance in the exchange. Finally, there is nothing to indicate that the museum owner/purchasing agent assumed the risk of what occurred

How well did you know this?
1
Not at all
2
3
4
5
Perfectly
15
Q

A landowner included in his will a provision giving “all of my property, both real and personal, wherever situated, to my widow for life, and after her death to any of our children who may survive her.”

What is the gift to the children?

A

Contingetn remainder — contingent upon surviving

How well did you know this?
1
Not at all
2
3
4
5
Perfectly
16
Q

A father executed a deed to his art gallery “to my daughter for her life, and on my daughter’s death to her children; provided, however, that if my daughter stops painting, to my brother.” The daughter has two children and is still painting.

At the time of the grant, what is the best description of the interest of the daughter’s two children?

A A contingent remainder.

B A vested remainder subject to open and to total divestment.

C A vested remainder subject to open.

D An executory interest.

A

The daughter’s two children have a vested remainder subject to open and subject to complete divestment. A remainder is a future interest created in a transferee that is capable of taking in possession on the natural termination of the preceding estate. A remainder is vested if the beneficiaries are ascertainable and their taking in possession is not subject to a condition precedent. A vested remainder created in a class of persons that is certain to take but is subject to diminution by reason of others becoming entitled to take is a vested remainder subject to open. Vested remainders may be subject to total divestment if possession is subject to being defeated by the happening of a condition subsequent. Here, the daughter’s two children have a remainder because, on the expiration of the daughter’s life estate, they will be entitled to possession of the property. The remainder is not subject to a condition precedent and the beneficiaries are in existence and ascertained, so the remainder is vested, not contingent. The remainder is subject to open because the daughter may have more children. Finally, the remainder is subject to total divestment because the daughter’s children’s right to possession is subject to being defeated by the daughter’s ceasing to paint.

How well did you know this?
1
Not at all
2
3
4
5
Perfectly
17
Q

An automotive engineer announced that he had developed a carburetor that will enable cars to achieve 100 miles per gallon of fuel, and that he will allow the carburetor to be inspected next month. Soon after, a former employer of the engineer brought an action to prohibit the engineer from displaying the carburetor, claiming that the engineer probably had stolen the carburetor’s design from the employer. The court granted the employer a temporary restraining order prohibiting the engineer from disclosing any mechanical details of his carburetor, and ordered a hearing to be held in one week to determine whether a preliminary injunction should be issued. Because each party would have to reveal the mechanical details of his designs at the hearing, the employer requested that the hearing be closed to the public and that the record be sealed to avoid revelation of his designs. The court granted the request. A reporter for a monthly automobile magazine heard about the case and wanted to attend the hearing. When he was told that the hearing would be closed, he filed an action to have it opened.

What is the reporter’s best argument for opening the hearing?

A Closure is not necessary to preserve an overriding interest here.

B The right of freedom of the press is extensive and allows the press to attend all hearings of interest to the public.

C Closure here amounts to a prior restraint.

D Under the fairness doctrine, the magazine will be required to give each litigant an opportunity to present his side of the case.

A

The reporter’s best argument is that the closure here is not necessary to preserve an overriding interest because trials and pretrial hearings generally must be open to the public. The Supreme Court has held, at least in the context of criminal cases, that trials and pretrial proceedings can be closed only if closure is necessary to preserve an overriding interest and the closure order is narrowly tailored to serve the overriding interest. While the Court has not yet established the standard for civil matters such as the case here, several Justices and commentators have suggested that the same standard will be applied in civil cases since they too have historically been open to the public.

How well did you know this?
1
Not at all
2
3
4
5
Perfectly
18
Q

While fleeing from an armed robbery he had just committed, a man struck a pedestrian with his car, seriously injuring the pedestrian. The robber was soon apprehended and charged with armed robbery and reckless driving, both felonies. Just prior to trial, the pedestrian died from his injuries. The trial on the robbery and driving charges proceeded, and the robber was convicted of the armed robbery charge and acquitted of the reckless driving charge. The robber was then indicted under the jurisdiction’s felony murder statute for causing the death of the pedestrian during the course of committing an armed robbery. The robber moved to dismiss the indictment on the ground that a second trial would violate double jeopardy.

Is the robber’s claim correct?

A Yes, because he was acquitted of the reckless driving charge.

B Yes, because the pedestrian died before the robber’s first trial had begun.

C No, because he was convicted of the armed robbery charge.

D No, because felony murder requires proof of an additional element not required by the felony itself.

A

The robber’s claim is correct because the victim died before jeopardy attached for trial on the lesser included offense. The Fifth Amendment right to be free of double jeopardy provides that once jeopardy attaches for an offense, the defendant may not be retried for the same offense. Under the Blockburger test, two crimes do not constitute the same offense if each crime requires proof of an additional element that the other crime does not require. Under this test, a lesser included offense and the greater offense would be considered the “same offense,” because the lesser included offense consists entirely of some, but not all, elements of the greater crime. Hence, under double jeopardy rules, attachment of jeopardy for the greater offense bars retrial for lesser included offenses, and attachment of jeopardy for a lesser included offense generally bars retrial for the greater offense.

An exception to this latter rule exists if all of the elements for the greater offense had not occurred at the time of prosecution for the lesser offense, but in this case the final element for the felony murder charge—the death of the victim—occurred before jeopardy had attached in the first trial, so the prosecution could have added a charge of felony murder prior to proceeding with the first trial. Thus, the underlying felony of armed robbery was a lesser included offense of the felony murder and the robber’s being placed in jeopardy for it bars the subsequent trial for the felony murder.

How well did you know this?
1
Not at all
2
3
4
5
Perfectly
19
Q

On April 10, the owner of a small farm mailed a letter to a new resident of the area who had expressed an interest in buying the farm. In this letter, the farm owner offered to sell the farm to the resident for $100,000. The offer expressly stated that the offer expires on June 1, “if acceptance by the offeree has not been received by the offeror on or before that date.”

On the morning of June 1, the resident sent a written acceptance to the farm owner by messenger. However, through negligence of the messenger company, the acceptance was not delivered to the farm owner until June 2. On June 4, the farm owner entered into a contract to sell the farm to another buyer for more money but did not inform the resident of the transaction. When the resident followed up by phone on June 10, the farm owner told him that he had sold the farm to another buyer.

Which of the following is the most correct statement?

A No contract between the farm owner and the resident arose on June 2.

B An enforceable contract arose on June 1.

C The farm owner’s silence constituted an acceptance of the resident’s message on June 2.

D A voidable contract arose on June 1.

A

Mailbox rule does not apply if contract says otherwise

No contract arose on June 2 because the farm owner’s offer expired on June 1, when the farm owner did not receive the resident’s acceptance. If a period of acceptance is stated in an offer, the offeree must accept within that period to create a contract. Failure to timely accept terminates the power of acceptance in the offeree (i.e., a late acceptance will not be effective and will not create a contract). Under the mailbox rule, an acceptance generally is effective upon dispatch (i.e., the acceptance creates a contract at the moment it is mailed or given to the delivery company). However, the mailbox rule does not apply where the offer states that acceptance will not be effective until received. In the latter case, acceptance is effective only upon receipt. Here, the farm owner’s offer specifically stated that the acceptance must be received by June 1 to be effective. Thus, the farm owner opted out of the mailbox rule, and no contract was created by delivery of the acceptance on June 2. Note that the resident will not be able to successfully argue that the acceptance was valid because the late delivery was the messenger company’s fault. This would be a valid argument if the mailbox rule applied here, because the acceptance would have been effective on June 1, when the message was given to the messenger company. However, by opting out of the mailbox rule, the farm owner put the burden of any negligence in delivery on the resident. Thus, there was no valid acceptance

How well did you know this?
1
Not at all
2
3
4
5
Perfectly
20
Q

Dramshop act

A pedestrian was struck and seriously injured by a car driven by an intoxicated driver. The driver had been served several alcoholic drinks by a bartender at a local bar. The pedestrian sued the bartender in a jurisdiction that does not have a dramshop act.

Is the bartender vicariously liable for the pedestrian’s injuries?

A No, because the driver acted recklessly by driving while intoxicated.

B No, because there is no dramshop act in the jurisdiction to impose liability.

C Yes, because there is no dramshop act in the jurisdiction to limit liability.

D Yes, because the intoxicated driver caused the pedestrian to suffer personal injuries.

A

Because the jurisdiction does not have a dramshop statute, the bartender will not be liable for the injuries caused to the pedestrian by the intoxicated driver. At common law, no liability was imposed on vendors of intoxicating beverages for injuries resulting from the vendee’s intoxication, whether the injuries were sustained by the vendee or by a third person as a result of the vendee’s conduct. Many states, in order to avoid this common law rule, have enacted “dramshop acts.” Such acts create a cause of action in favor of any third person injured by the intoxicated vendee. Without a dramshop act, the bartender will not be vicariously liable. (A) is incorrect because it implies that the bartender would be vicariously liable if the driver was not reckless. Without a dramshop act, however, there can be no vicarious liability imposed on the bartender regardless of whether the driver’s actions are characterized as reckless or simply negligent.

How well did you know this?
1
Not at all
2
3
4
5
Perfectly
21
Q

The President of the United States issued an executive order banning conversation by all executive employees with members of the press unless prior permission had been obtained from a supervisor. Executive Department employees were subject to dismissal for violation of the order. An employee of the United States Department of Agriculture spoke to a reporter and told the reporter that the USDA sanctioned the approval of a dangerous pesticide because of payoffs made by lobbyists to a high-ranking USDA official. The newspaper printed the story and quoted the employee by name. After a hearing in line with civil service regulations, the employee was found to have violated the executive order barring unauthorized conversations with news media reporters, and the employee was fired. The employee brought suit in federal court for reinstatement, back pay, and other benefits and also prayed that the court should strike down the executive order as unconstitutional.

If the employee’s case reaches the United States Supreme Court, how should the Court rule?

A The executive order is constitutional, because the President has plenary power to control Executive Department employees.

B The executive order is constitutional, because government employment is a privilege and not a right.

C The executive order is unconstitutional, because Congress, rather than the President, has authority to set the terms of federal employment.

D The executive order is unconstitutional, because the President cannot broadly limit all executive employees’ freedom of speech and association.

A

The President’s order is unconstitutional because it is too broad a limit on the freedom of speech and association of government employees. No government employees were allowed to have even a conversation with the press without the permission of a supervisor. Thus, it is unconstitutional. (A) is incorrect because the President’s plenary power to control executive employees is limited by the Constitution. For example, the President cannot violate the First Amendment (or other constitutional guarantees) in regulating federal employees. (B) is incorrect because the Court no longer draws a distinction between rights and privileges. Furthermore, government employees would retain their First Amendment rights even if government employment were termed a “privilege.” (C) is incorrect because the President has authority to enact some regulations for federal employees. The problem with this regulation is that it unduly restricts freedom of speech.

How well did you know this?
1
Not at all
2
3
4
5
Perfectly
22
Q

P has introduced part of a record. Can D ask P to produce the rest?

A

YES.
(B) The defendant may compel the plaintiff to introduce the remainder of the record. When part of an act, conversation, or writing is introduced into evidence, the Federal Rules provide that the adverse party may compel the proponent of the evidence to introduce any part thereof that ought, in fairness, to be considered at the same time. Here, the remainder of the shipping record that supports the defendant’s position should be introduced at the same time as the part introduced by the plaintiff. Answer (A) is incorrect because no foundation need be laid for another part of a record that has already been introduced

How well did you know this?
1
Not at all
2
3
4
5
Perfectly
23
Q

RAP time!

A landowner’s will left his ranch to a rancher, his heirs, and assigns, so long as the property was used exclusively for ranch purposes, then to the landowner’s grandson. The remainder of the landowner’s property passed through the residuary clause of his will to the grandson. Seven years after the landowner’s death, the rancher began strip mining operations on the ranch. The grandson brought an action to quiet title to the ranch against the rancher, and the rancher counterclaimed on the same theory.

Who should prevail?

A The rancher, because the condition imposed on his interest under the will is void as violating the Rule Against Perpetuities.

B The rancher, because the condition imposed is a restraint against alienation.

C The grandson, pursuant to the residuary clause.

D The grandson, because the condition imposed is valid and he takes according to the subsequent provision.

A

The grandson prevails because the ranch passed through the residuary clause. Under the Rule Against Perpetuities, the attempt to give the grandson an executory interest is void, so (D) is incorrect. However, the courts would simply read the conveyance without the language of the executory gift, leaving a possibility of reverter in the grantor, the landowner. Thus, (A) is incorrect. Because the grandson succeeded to the landowner’s interest as grantor via the residuary clause of the will, he will prevail. (B) is incorrect because there is no restraint on alienation contained in the will

24
Q

A landowner owns 15 acres of undeveloped property. He plans to build a stadium complex on the property to house a football team two years from now, but would like to open the 15 acres to public use for picnicking and similar activities until then.

Which of the following would best accomplish the landowner’s goal?

A Dedicate the 15 acres for use as a public park.

B Lease the 15 acres to the city for two years.

C Grant the city an easement for public recreational uses for two years.

D Covenant that the city may use the 15 acres for recreation for two years.

A

The best way for the landowner to accomplish his goals is to grant the city an easement for recreational use for two years. An easement would allow the city to use the land only for the purposes provided for in the easement, and the landowner could limit the purposes to recreational uses.

D) is not a good choice because covenants usually are made in conjunction with a lease, deed, or other instrument; they promise some act or forbearance with respect to property and are generally not used to grant rights for access to property.

25
Q

A police officer saw a car containing three teenagers driving slowly down the street at 1 a.m. She waited for it to go by her and, after it was far enough ahead, started to follow it. Several blocks later, the car rolled through a stop sign. The officer immediately pulled the car over and requested the driver’s license. A license check showed that the driver had five outstanding parking tickets. A statute in the jurisdiction permits an arrest to be made if a driver has four or more outstanding parking or traffic violations. The officer decided to take the driver in on the tickets. She informed the driver that he was under arrest and asked him to step out of the car. When the driver got out, the officer patted him down and found a gun in his waistband. Calling for backup, she decided to haul all three teenagers to jail.

Subsequent testing showed that the gun had been used in a recent homicide during a store robbery by three young men. One of the passengers made a motion to prevent the introduction of the gun at his trial for murder and robbery.

How should the judge rule?

A Deny the motion, because the gun was found after the driver had been arrested.

B Deny the motion, because the officer lawfully stopped the car.

C Grant the motion, because the officer had no valid reason to be following the automobile.

D Grant the motion, because the officer had not arrested the driver for suspicion of robbing the store or committing the homicide.

A

The judge should deny the motion. Evidence will be suppressed if it was obtained in violation of the defendant’s constitutional rights. Each passenger in a car has standing to challenge a stop of the car. If a stop is invalid, under the fruit of the poisonous tree doctrine, evidence obtained as a result of the invalid stop will be suppressed. Here, the officer had a valid reason to stop the car. An officer may stop a car for violating a traffic law, and here the driver of the car failed to stop at a stop sign.

(A) is incorrect because it is irrelevant. A person may seek suppression of evidence that has been seized only if the seizure is in violation of the person’s own constitutional rights. While it is true that the gun was properly seized from the driver because the arrest appears to have been valid, the gun would be admissible against the passenger even if it had been unlawfully seized from the driver. The driver, of course, would have standing to complain of the unlawful seizure, but not the passenger.

26
Q

The homeowner contacted a qualified repairman to find out how to fix the problem and was told it would cost an additional $300 for new blowers and $150 in labor costs to replace the faulty blowers. The homeowner mailed the heating company a copy of the repair estimate and a check for $3,050—the contract price less the cost of new blowers and labor to install them—and wrote prominently on the check “Payment in full for installation of two baseboard heaters.” The heating company cashed the check upon receipt. The heating company then sued the homeowner for $450, the difference between the agreed contract price and the amount paid.

Is the heating company likely to prevail in its suit seeking the $450 from the homeowner?

A No, because the heating company failed to fix the problem after several attempts.

B No, because the heating company cashed the check.

C Yes, because the check did not represent payment in full (i.e., the contract price) for the work done.

D Yes, because the heating company should have been given more time to cure the defect.

A

Is this a successful accord and agreement?

YES ;cause you cashed the check.

27
Q

A state statute provided for criminal penalties for “knowingly selling alcoholic beverages in violation of the regulations of the State Liquor Commission to any person under the age of 18.” One of the State Liquor Commission regulations provided that “before an alcoholic beverage is sold to any person between the ages of 17 and 24, the seller must demand some form of photo identification to determine the buyer’s age.”

A minor who looked much older than his age of 17 walked into a tavern located in the state and asked the bartender for a beer. The bartender never asked the minor for any form of identification, as he thought that he was at least 25 years old. Had the bartender asked for identification, the minor would have shown him a fake identification card showing that he was 21 years old. The bartender served the beer to the minor, who consumed it on the premises. The bartender was subsequently charged under the state statute for selling the beer to the minor.

Is the bartender guilty?

A No, because he reasonably believed that the minor was older than 25 years.

B No, because the minor had fake identification with which he could have obtained the beer.

C Yes, because he sold an alcoholic beverage to a minor, a strict liability crime.

D Yes, because he failed to ask for identification, and the regulation does not provide for a mens rea requirement.

A

The bartender’s reasonable belief that the minor is 25 years old is a mistake of fact that negates the state of mind required by the statute. Ignorance or mistake as to a matter of fact will affect criminal guilt only if it shows that the defendant did not have the state of mind required for the crime. In addition, the mistake must be reasonable unless the offense is a specific intent crime. Here, the statute requires that the defendant have acted “knowingly” with respect to each of the material elements of the offense. A person acts knowingly with respect to the nature of his conduct when he is aware that his conduct is of that nature or that certain circumstances exist. At least one of the material elements of the offense here is that the sale be to a person under the age of 18. If the bartender believed that the minor was 25 years old, the bartender has not acted knowingly with respect to the fact that the purchaser was under 18, and he cannot be convicted of violating the statute.

C) is incorrect. Although some states may make selling liquor to minors a strict liability crime, the state here has added a “knowingly” requirement that must be satisfied beyond a reasonable doubt for a conviction.

28
Q

In a products strict liabiltiy case, should manufacturers put warning for everything?

A

NO only for things they reasonably foresee

Knowledge on the part of the manufacturer that its television was being mounted on the ceiling would give rise to a duty to include in the manual warnings against the practice or detailed instructions on how to safely mount it. The television hardware and instructions were appropriate for its intended mounting on the wall. However, courts in a strict liability case require a commercial supplier to anticipate reasonably foreseeable uses even if they are misuses of the product. If the manufacturer knew that members of the public were sometimes mounting the television on the ceiling, marketing the product without including either warnings against the practice or appropriate hardware and instructions on how to safely do so made the product so defective as to be unreasonably dangerous if it were improperly mounted. Under a strict liability theory, the manufacturer is liable for supplying a defective product. As a guest of a purchaser of the product, the woman is a foreseeable plaintiff; thus, the manufacturer may be liable to her. The defective product actually and proximately caused the woman to suffer serious injuries. Therefore, the manufacturer is liable to the woman in a strict products liability action.

29
Q

Congress imposes taxes on smoking companies that goes to pay anti smoking ads. Against 1st amendment?

A

NO. Although people (and corporations) cannot generally be forced to convey a message with which they disagree, they can be forced to pay taxes that fund messages with which they disagree.

30
Q

What does one party have to give to the other party without any discovery request?

A

Without waiting for a discovery request, a party must provide to the other parties copies or descriptions of documents that are in the disclosing party’s possession or control and that the disclosing party may use to supports its claims or defenses

31
Q

A landlord leased residential property to a tenant. The written lease was for a period of one year, with the monthly rent of $1,000 payable on or before the first of each month. The termination date set out in the lease was October 1. On August 10 of the first year of her tenancy, the tenant received a letter from the landlord along with a new lease form. The lease was for a period to terminate on October 1 of the following year, and the rent stated in the new lease was $1,200 per month. Both the rent increase and the notice given were in full compliance with relevant state statutes. An accompanying letter, signed by the landlord, asked the tenant to sign the lease on the line marked “tenant.” On September 15, the tenant sent the lease back to the landlord unsigned. On September 20, the tenant sent a letter to the landlord by certified mail. The landlord signed the return receipt, which the post office duly sent to the tenant. Enclosed with the tenant’s letter was a check for $1,000 for “next month’s rent.” The landlord deposited the check into his bank account. With the landlord’s acquiescence, the tenant remained in possession after October 1.

Which of the following statements is most accurate?

A The tenant has a month-to-month tenancy at $1,000 monthly rent.

B The tenant has a month-to-month tenancy at $1,200 monthly rent.

C The tenant has an annual tenancy at $1,200 per month rent.

D The tenant has a tenancy at will.

A

The tenant has a month-to-month tenancy at $1,200 per month. When a tenant continues in possession after termination of her right to possession, the landlord may bind the tenant to a new periodic tenancy. While the terms and conditions of the expired tenancy generally apply to the new tenancy, if the landlord notifies the tenant before termination that occupancy after the termination date will be at an increased rent, the tenant will be held to have acquiesced to the new terms if she does not surrender. This is so even if the tenant objects to the increased rent, as long as the rent increase is reasonable.

32
Q

WTF question

A buyer entered into a contract with a seller to purchase the seller’s farm. The contract of sale referred to the farm as containing 250 acres. The agreed-on price was $1 million. Before the date on which escrow was to close, the buyer learned from a surveyor he had hired that the farm actually contained 248 acres. On the date the sale was to close, the buyer instructed the escrow agent to release all but $8,000 of the purchase money because he was not getting what he bargained for. The seller refused to proceed with the sale. The buyer brings an action for specific performance and also seeks an $8,000 reduction of the agreed-upon contract price.

What will be the probable outcome of the litigation?

A The seller will win, because the buyer refused to tender the contract price when the seller tendered substantially what the contract called for her to perform.

B The seller will win, because both parties had seen the farm before the contract was formed.

C The buyer will win, because he is not receiving what he bargained for under the contract.

D The buyer will win, if the court finds that the $8,000 reduction in price is a fair reflection of the title defect.

A

This answer states the traditional rule where the amount of land in a land sale contract is less than as agreed. When a buyer has a remedy of specific performance in a land sale contract, a court of equity will order a seller to convey the title if the buyer tenders the purchase price. If the seller cannot provide marketable title under the terms of the contract, but the buyer wishes to proceed with the transaction, the buyer can usually get specific performance with an abatement of the purchase price in an amount reflecting the title defect. A defect as to the quantity of land conveyed is usually corrected by a pro rata abatement of the price.

(D) states the factors that a court of equity will look for when deciding whether to grant specific performance with abatement.

(A) is incorrect because the parties’ contract did not merely refer to the farm as a named parcel of land; it recited that it contained 250 acres. Based on this recital, a court could readily conclude that the difference of two acres is a material change in the terms of the contract and that the seller’s tender of 248 acres was not substantial performance.

33
Q

A township located in a farming community was composed mostly of persons belonging to a specific religious sect. To help instill proper respect for authority in children, which was a central tenet of the sect, and to maintain order in the classroom, the local school board allowed teachers to inflict corporal punishment. Such punishment was inflicted on a fourth grader in a township school immediately after his teacher saw him pulling a girl’s hair. Neither he nor his parents belonged to the religious sect. When the boy’s parents learned of the incident, they hired an attorney. Rather than suing the teacher for battery as permitted under state law, the attorney brought an action against the teacher under a federal statute providing a cause of action for damages against any government employee who deprives a person of his constitutional rights.

Should the court find the policy allowing corporal punishment to be constitutional?

A No, because the punishment policy violates the First Amendment Establishment Clause.

B No, because the boy was denied any kind of hearing, in violation of his right to procedural due process under the Fourteenth Amendment.

C Yes, because under the doctrine of parens patriae states may impose any punishment they see fit.

D Yes, because the punishment was not grossly disproportionate under the Eighth and Fourteenth Amendments.

A

The punishment here is constitutional because it does not violate any constitutional provision. The best answer reflecting this reasoning is

(D)-there was no Eighth Amendment violation here-because paddling students as a disciplinary measure has not been found to be cruel and unusual punishment.
(A) is incorrect because there is no Establishment Clause violation here. Under the Establishment Clause, if there is no sect preference, government action generally will be upheld if the action serves a secular purpose, its primary effect neither advances nor inhibits religion, and it does not excessively entangle government with religion. There is no sect preference under the school board’s corporal punishment rule here, the rule has the secular purpose of maintaining order in the classroom (the fact that this coincides with the tenets of a local religion does not change that conclusion), its main purpose neither advances nor inhibits religion, and there is no excessive entanglement.

(B) is incorrect because there has been no deprivation of procedural due process. The Supreme Court has held that although corporal punishment may involve a liberty interest, no hearing is required prior to inflicting such punishment; the possibility of a common law action in tort is sufficient procedural protection. [Ingraham v. Wright (1977)]

34
Q

The United States Surgeon General was cited for contempt for refusing to answer questions as part of a Senate investigation regarding an issue in the Food and Drug Administration.

His contempt citation will be dismissed if he can show which of the following?

A As a member of the executive branch, he is immune from prosecution.

B If he answered the questions, he could be subject to dismissal from his position as Surgeon General.

C The questions do not relate to any matter concerning which the Senate may legislate.

D The questions do not relate to any matter concerning current or planned legislation.

A

His contempt citation will be dismissed if he can show that the questions do not relate to any matter concerning which the Senate may legislate. Congress’s power to investigate is limited to matters on which it can legislate. Therefore, if the Surgeon General can demonstrate that the questions concerned matters upon which Congress could not legislate (not an enumerated power under Article I, Section 8), then this contempt citation must be dismissed.

(B) is wrong because he would have a privilege not to answer only if he is subject to criminal liability. Merely because he may get fired is not sufficient grounds for him to refuse to answer a lawful question posed by a member of the Senate in an appropriate hearing. (

35
Q

A 13-year-old boy who lived on a farm with his parents in a rural area had learned to drive the family’s tractor when he was 11. A state statute permitted persons without a driver’s license to operate farm vehicles on public roads for short distances. One morning the boy took the tractor onto a public road to reach one of the outlying fields a few hundred yards away. As he neared the field he was distracted by a girl riding by on a bicycle, and cut in front of a milk delivery truck that was starting to pass him. The truck swerved off the road, injuring the driver.

If the driver sues the boy to recover damages for his injuries, which of the following statements is most correct regarding the standard of care to be applied?

A The state statute replaces the general common law standard of care with a statutory standard.

B The trier of fact should take into account the boy’s experience at driving a tractor when considering the applicable standard of care.

C Persons 13 years of age or older are held to the same standard as adults.

D An adult standard of care will not be applied because it is common in that region for children of that age to be operating tractors.

A

The most correct statement is that the trier of fact should take into account the boy’s experience when considering the applicable standard of care. Regardless of the specific standard of care that is applied, someone with knowledge superior to that of the average person is required to use that knowledge. Hence, the trier of fact should take into account the fact that the boy had driven a tractor since he was 11 years old.

(A) is incorrect. The precise standard of care in a common law negligence case may be established by proving the applicability to that case of a statute providing for criminal penalties, so that the statute’s specific duty will replace the more general common law duty of due care, and a violation of the statute will establish duty and breach of duty. Here, nothing in the facts indicates that the boy violated any provisions in the statute, but he still may be liable to the truck driver for breach of a general duty of care.

(C) is incorrect. The usual standard of conduct to which a child must conform is that of a child of like age, education, intelligence, and experience. While a child must conform to an adult standard of care when engaging in a potentially dangerous activity in which usually only adults engage, there is no blanket rule that children 13 years of age or older are held to the same standard of care as adults.

(D) is incorrect because the fact that 13-year-olds commonly drive tractors in that region does not preclude the court from applying an adult standard of care when a tractor is driven on a public road.

36
Q

A consumer purchased a new television set from an electronics store. When he got home, he opened the box and found an owner’s manual that contained operation instructions, warnings regarding the danger of electricity, and a warranty that stated:

“The store expressly warrants that this set shall be free of manufacturing defects for 30 days. If a set is defective, the store’s liability shall be limited to the cost of repair or replacement of defective parts. The store “HEREBY DISCLAIMS ANY AND ALL OTHER WARRANTIES, EXPRESS OR IMPLIED, INCLUDING THE WARRANTY OF FITNESS FOR PARTICULAR PURPOSE AND THE WARRANTY OF MERCHANTABILITY.””

Five weeks later, after the set was properly installed, the consumer turned on the set, heard a crackling noise, and watched as his television exploded and was destroyed.

Under which of the following theories will the consumer most likely recover?

A Breach of express warranty.

B Breach of the implied warranty of merchantability.

C Breach of the implied warranty of fitness for a particular purpose.

D Breach of the warranty of reasonable workmanship.

A

Disclaimer question

The consumer will most likely recover for breach of the implied warranty of merchantability. In every sale of goods, unless expressly disclaimed, there arises a warranty that the goods will be merchantable, which means that they will be fit for the ordinary purposes for which such goods are used. A television that explodes after five weeks of use likely breaches this warranty. The warranty will not be disclaimed because, to be effective, a disclaimer must be part of the offer and acceptance process or must be agreed to by the buyer as a modification. Here, the “disclaimer” was in the box, and the consumer did nothing to accept the disclaimer. Thus, (B) is correct. (A) is wrong because no express warranty was made as to quality at a time sufficient to become part of the bargain. An express warranty will arise from any affirmation of fact or promise made by the seller to the buyer, or from any description of the goods, and any sample or model if the statement, description, sample, or model is part of the basis of the bargain. Here, there was no such affirmation, promise, description, sample, or model indicating that the television would not explode; that is something that would be understood (implied).

37
Q

A feed store owner agreed to purchase several tons of grain products at a specified price from a large supplier of cattle feeds. The supplier later failed to deliver the promised grains, and the owner was forced to cover by purchasing from local producers at a higher price. The owner contacted a large law firm in the city and obtained their agreement to represent him in connection with his possible claims against the supplier. Due to error, the applicable statute of limitations period passed without the filing of any action on the owner’s behalf. The owner retained another lawyer and sued the large law firm for malpractice. The jurisdiction retains traditional contributory negligence.

In addition to the firm’s negligence, what else does the owner have to establish as part of his prima facie case?

A He had a good faith claim against the supplier that was lost by the law firm’s dilatoriness.

B He would have recovered from the supplier if an action had been timely filed.

C He did not contribute to the failure to timely file an action through his own negligence.

D The losses resulting from breach of the sales agreement by the supplier severely harmed his financial situation.

A

The owner will have to show that he would have recovered damages in his lawsuit. The following elements must be proved for a prima facie case of negligence: (i) the existence of a duty on the part of the defendant to conform to a specific standard of conduct for the protection of the plaintiff against unreasonable risk of injury, (ii) breach of that duty by the defendant, (iii) that the breach of duty was the actual and proximate cause of the plaintiff’s injury, and (iv) damage to the plaintiff’s person or property. Here, the owner can establish that the law firm breached its professional duty of care by failing to file a claim within the statute of limitations. He must also establish that this breach was an actual and proximate cause of his damages, which here would be the loss of the contract damages that he could have recovered from the breach by the supplier.

38
Q

Is opening an unlocked door sufficient for breaking in burglary?

A

Yes, the requirement of a breaking is satisfied as long as some degree of force is used to gain entry. Opening a closed door is sufficient for this element; there is no requirement that the door be locked.

39
Q

Adverse posession

A

It is important to note that an adverse possession need not necessarily be effective on an entire property. A person can successfully adversely possess a portion of the property as long as the possessor excludes the actual owner from that portion. In such a case, the adverse possession will succeed in transferring only the property that was actually possessed by the adverse possessor. For example:

Jerry owns a two bedroom apartment in a condominium on the west side of Manhattan. Because he lives alone, he only uses one of the bedrooms. One day, Kramer moves into the second bedroom, and he has a lock installed on the door to that second bedroom. He lives in that bedroom for the entire adverse possession period and, all the while, he does not allow Jerry to enter the bedroom in which he is living. This would be considered adverse possession, and Kramer would gain title to the second bedroom, while Jerry would keep title to the rest of the apartment.

40
Q

The plaintiff is a State A corporation that entered into a contract with the defendant under which the defendant agreed to manufacture and sell equipment to the plaintiff. The equipment is heavily regulated by the federal government. The plaintiff plans to sue the defendant for $1 million because the equipment does not conform to the contract specifications and does not operate properly. The defendant is incorporated in State B, but all of its facilities and offices are in State A.

Would a federal district court have subject matter jurisdiction over the plaintiff’s action?

A Yes, because the two corporations were incorporated in different states.

B Yes, because there is federal question jurisdiction.

C No, because the defendant voluntarily conducted business in State A.

D No, because all of the defendant’s facilities and offices are in State A.

A

The court does not have subject matter jurisdiction. Diversity of citizenship jurisdiction is available when: (i) there is complete diversity of citizenship, meaning that each plaintiff is a citizen of a different state from every defendant; and (ii) the amount in controversy exceeds $75,000. For diversity purposes, a corporation is considered to be a citizen of every state in which it was incorporated and the one state in which it has its principal place of business. In the instant case, the plaintiff is a State A corporation. The defendant corporation was incorporated in State B, but all of its facilities and offices, and presumably its headquarters, are in State A. Thus, the plaintiff corporation is a citizen of State A, while the defendant corporation is a citizen of both State A and State B. The State A-State A connection destroys complete diversity. (A) is incorrect because, as stated above, a corporation may have multiple citizenships for diversity purposes; the place of incorporation is just one citizenship for diversity purposes. (B) is incorrect. Federal courts do have subject matter jurisdiction when the plaintiff’s claim arises from federal law. However, the fact that the equipment is heavily regulated is not sufficient to raise a federal question. The cause of action is still a state law breach of contract claim. (C) is incorrect. Contacts with a state are used to determine whether a court has personal jurisdiction over a particular defendant. Contacts are not used to determine whether there is subject matter jurisdiction. Thus, the fact that the defendant voluntarily conducted business in State A is irrelevant.

41
Q

A landowner conveyed his land “to my son for life, then to my son’s widow for her life, then to my son’s children.” At the time of the conveyance, the son was 20 years old and unmarried. The son eventually married and had two children, the landowner’s grandson and granddaughter.

Many years later, the landowner and the grandson were involved in a train accident. The landowner was killed instantly. The grandson died a short time later of his injuries. The landowner left his entire estate by will to his friend. The grandson’s will devised his entire estate to the city zoo. The son’s wife was so grief-stricken that she became ill and died the next year, leaving her entire estate to her husband.

Eventually the son met and married a 21-year-old. Ten years later, the son died, leaving everything to his second wife. When the second wife moved onto the land, the granddaughter filed suit to quiet title to the land, joining all of the appropriate parties.

If the jurisdiction recognizes the common law Rule Against Perpetuities, unmodified by statute, in whom will the court most likely find that title to the land is held?

A One-half in the granddaughter and one-half in the city zoo, subject to the second wife’s life estate.

B One-half in the granddaughter and one-half in the second wife, because the second wife took the son’s interest.

C Entirely in the friend, subject to the second wife’s life estate, because the gift to the son’s children violates the Rule Against Perpetuities.

D Entirely in the granddaughter, subject to the second wife’s life estate, because the grandson did not survive the son.

A

The granddaughter and the city zoo each own one-half of the land, subject to the second wife’s life estate. At the time of the conveyance by the landowner, the son had a life estate, the son’s widow had a contingent interest (because the son’s “widow” cannot be ascertained until the son’s death), and the son’s children had a contingent remainder (because they have not yet been born). When the grandson and the granddaughter were born, however, their interests became vested subject to open (i.e., if the son had more children). Thus, when the grandson died, he had a vested remainder subject to open that he was free to devise by will; the city zoo took his vested remainder subject to open. At the son’s death, the class of his “children” closed (because the son could not have any more children), and the granddaughter’s and the zoo’s vested remainders subject to open became indefeasibly vested. Also at the son’s death, his widow was ascertained and her interest vested in possession. Because the second wife was the son’s widow, she is entitled to the valid life estate. Thus, the granddaughter and the city zoo hold one-half interests, subject to the second wife’s life estate.

(B) is wrong because the son had no interest in the land when he died. He merely had a life estate, which ended at his death. He did not inherit any interest in the property from anyone else. The only person he inherited from in these facts was his first wife, and she had no interest in the land. Furthermore, this choice overlooks the city zoo’s interest, which was inherited from the grandson.

(C) is wrong because the son’s children’s interest does not violate the Rule Against Perpetuities. To be valid under the Rule, an interest must vest if at all within a life in being at its creation plus 21 years. The son is a life in being. At the son’s death, his children’s interest is certain to vest or fail: If the son had any children, at his death, the children’s interest would become indefeasibly vested (i.e., the class would close and the children’s interest would no longer be subject to open). Note that the children need not come into possession within the perpetuities period; the only requirement is that their interests vest within the period. Likewise, if the son had no children, the gift to them was certain to fail at his death. Thus, the children’s interest does not violate the Rule. Because the son had children and their interest was valid, there was no interest to revert to the landowner and to be devised to the friend. Note that the unborn widow aspect of this question is a red herring. The fact would be relevant only if the children’s gift were conditioned on their surviving the widow, in which case the takers would remain unascertained and their interest would remain contingent until that time. But because the children’s interest vested at the son’s death, it is irrelevant that the son’s “widow” was not a life in being at the creation of the interest.

42
Q

A landowner was declared insane and committed to a state mental hospital 30 years ago. Five years after that, a trespasser entered onto the landowner’s 200-acre parcel of land, which was enclosed by a barbed wire fence. A solid wood fence ran through the middle of the land, separating the property into approximately equal east and west parcels. The trespasser began grazing cattle on the west parcel; no one else has been in possession of any part of the 200 acres. The period of time to acquire title by adverse possession is 15 years. Thirteen years ago, the landowner was declared competent and released from the hospital, but he did nothing until this year, when he brought an action to eject the trespasser. The trespasser counterclaimed to quiet title in him.

In this action, will the trespasser likely prevail?

A Yes, because he has acquired title to the 200 acres by adverse possession.

B Yes, but only as to the west parcel, because that is the portion of the land he actually occupied during his adverse possession.

C No, because the landowner can assert the defense of laches, as the trespasser did not bring an action to quiet his title within a reasonable time after the statute had run.

D No, because the landowner was insane for 12 of the 25 years that the trespasser was in possession.

A

The trespasser will lose because the landowner’s insanity delayed the running of the statute of limitations. The insanity disability prevents the running of the statute of limitations for adverse possession if the disability existed on the day the adverse possession began. Because the landowner was insane when the trespasser began the period of adverse possession, the clock did not begin to run until the landowner was free of the disability 12 years later; thus, the clock has run 13 years to the present. That is not enough to satisfy the statute, so the trespasser gets nothing.

43
Q

In litigation over whether an uncle conveyed a parcel of land to his nephew, the nephew wishes to offer into evidence a tape recording of his uncle made by a well-known oral historian at the nearby state university. The voice on the tape is discussing various conveyances of the parcel of land and other property owned by the uncle. The nephew wishes to have the historian testify that the voice on the tape is the uncle’s.

If the court allows the historian to testify, it will be because:

A The historian is testifying regarding an admission by a party-opponent.

B The historian has heard the uncle speak before.

C The historian became familiar with the uncle’s voice before the dispute over the property arose.

D The historian’s experience as an oral historian qualifies him as an expert in voice recognition.

A

The court will allow the historian to testify as to the identity of the voice simply because he is familiar with the uncle’s voice. Where the identity of a speaker is important, the oral statements require authentication as to the identity of the speaker. A voice, whether heard firsthand or through a tape recording, may be identified by the opinion of anyone who has heard the voice at any time. As long as such a foundation is laid to show familiarity with the voice, a lay opinion as to the identity of the speaker is permissible. Thus, because the historian became familiar with the uncle’s voice when he made the tape recording, he will be permitted to testify that the voice on the tape was the uncle’s. (A) is incorrect because, even assuming that the uncle is a party opponent, the historian is only testifying as to the identity of the speaker rather than any admissions that the uncle may have made.

44
Q

A decedent died without having executed a will, leaving a substantial estate to be distributed by the probate court. The jurisdiction’s applicable statute provides that where a decedent leaves neither issue nor spouse, nor parents, his estate goes to his brothers and sisters and their descendants. The decedent was never married, had no children, and both of his parents are dead. A woman whose birth certificate was destroyed by fire seeks to establish that she is the daughter of the decedent’s only sibling, who is now also deceased. The woman offers into evidence a statement in a properly recorded trust instrument. The instrument was executed by the decedent’s father and recited that certain specified real property conveyed by the decedent’s father into the trust should be held for her benefit, as “my loving granddaughter.” The document actually offered is an enlarged print photocopy of microfilm records, authenticated by an employee of the county.

What should the trial court do?

A Exclude the evidence, because it is not the best evidence.

B Exclude the evidence, because it is inadmissible hearsay not within any recognized exception.

C Admit the evidence, because it is a record of a document affecting an interest in property.

D Admit the evidence, because it constitutes a past recollection recorded.

A

The court should admit the evidence. Statements in a document affecting an interest in property are admissible, pursuant to Federal Rule 803(15), if they are relevant to the purpose of the document.The builder will be discharged from his duties under the contract. Modern courts recognize that impracticability due to excessive and unreasonable difficulty or expense is a defense to breach of contract for nonperformance. Since the cost to the builder to perform under the original contract would exceed more than double what he would be paid, he likely would be excused from performance by commercial impracticability. (D) is therefore incorrect. Unlike destruction of the building itself before completion, which will not discharge a contractor’s duty, the erosion of the lot, which destroys the means of performing the contract, will generally not be one of the risks that a builder will be deemed to have assumed.

45
Q

While the defendant was committing a robbery, he shot and killed the victim. The defendant is charged with first degree murder in a state that defines first degree murder as murders committed with premeditation or deliberation or during the commission of burglary, arson, rape, or robbery, and defines second degree murder as all other murders. The state also defines voluntary manslaughter as the unlawful killing of a human being with malice upon a sudden quarrel or heat of passion, and it defines involuntary manslaughter as the unlawful killing of a human being without malice in the commission of an unlawful act, not amounting to an enumerated felony, or in the commission of a lawful act that might produce death in an unlawful manner or without due caution and circumspection.

Assuming evidence to support, what explanation for the shooting would best help the defendant in avoiding conviction for first degree murder?

A In an act of resistance, the victim suddenly attacked the defendant and knocked him down, so the defendant pulled the trigger because he was afraid the victim was going to hit him again.

B The defendant had the gun for many years, it was old and rusty, and he did not think it would fire.

C The defendant had taken “angel dust” before the incident and does not remember getting a gun or holding up the victim.

D When the defendant tried to hold up the victim, the victim said, “Get out of here, you dirty bum, or I’ll kill you,” and the defendant became so upset that he did not know what he was doing.

A

Because the defendant was charged with first degree murder, the theory of the case is most likely felony murder, and

(C) is the only choice that sets out a theory to avoid a felony murder conviction. If the defendant was so intoxicated that he could not form the intent to steal, then he is not guilty of robbery, and there would be no “felony” from which the felony murder rule is to arise. (A) and

(B) are incorrect because even though the defendant could argue that no premeditation or deliberation was present, he would still be guilty of first degree murder under the felony murder rule because the felony was robbery.

(D) is wrong because insulting someone is not adequate provocation that would mitigate a homicide to voluntary manslaughter; neither would this “threat” suffice, in all likelihood. At most, the circumstances might produce the sort of unreasonable anger that would negate premeditation and deliberation. However, because the defendant clearly caused the homicide while committing the felony of robbery, his crime remains first degree murder.

46
Q

Acting with probable cause, the police arrested a man in connection with the armed robbery of a liquor store. After being given Miranda warnings, the man confessed to the robbery but denied his involvement with several other recent armed robberies of businesses in the area. He was formally charged with the one robbery and put into a cell with a paid informant working undercover for the police. The informant had been instructed to find out what he could about the other robberies but not to ask any questions. The informant began talking about a convenience store robbery in which a bystander was shot and seriously injured by the robber, and he deliberately misstated how it happened. The man, unaware that his cellmate was an informant, interrupted to correct him, bragging that he knew what really happened because he was there, and proceeded to make incriminating statements about the robbery. The man was subsequently charged with armed robbery and attempted murder in the convenience store robbery.

At a motion-to-suppress hearing on that charge, if the man’s attorney moves to exclude the statements made to the informant, should the motion be granted?

A Yes, because the informant deliberately elicited incriminating statements in violation of the man’s Sixth Amendment right to counsel.

B Yes, because the informant’s conduct constituted custodial interrogation in violation of the man’s Fifth Amendment privilege against self-incrimination.

C No, because the man had not yet been charged with the robbery of the convenience store when he made the statements to the informant.

D No, because the informant’s conduct did not constitute interrogation.

A

The man’s motion should be denied because neither his Fifth nor Sixth Amendment rights were violated by the informant’s conduct. The Sixth Amendment right to counsel applies to all critical stages of a criminal prosecution after formal proceedings have begun, but does not apply in precharge custodial interrogations. Because this right is “offense specific,” the fact that the right to counsel has attached for one charge does not bar questioning without counsel for an unrelated charge. Because the man has not been charged with the convenience store robbery, his Sixth Amendment right to counsel has not been violated. The Fifth Amendment privilege against self-incrimination requires Miranda warnings and a valid waiver before any statement made by the accused during custodial interrogation can be admitted. However, this requirement does not apply where interrogation is by an informant who the defendant does not know is working for the police, because the coercive atmosphere of police-dominated interrogation is not present. [Illinois v. Perkins (1990)] Because the man was not aware of the informant’s status, the informant’s conduct did not constitute a police interrogation.

(A) is wrong despite the fact that the informant’s conduct may have been deliberately designed to elicit incriminating remarks. As discussed above, the man’s right to counsel did not attach for purposes of the convenience store robbery

D) is incorrect because interrogation refers not only to express questioning, but also to any words or actions on the part of the police that the police should know are reasonably likely to elicit an incriminating response from the suspect. Here, the informant, working for the police, made statements about the convenience store robbery that were intended to, and reasonably likely to, prompt a response from his cellmate. Hence, it is not the absence of “interrogation” that avoids the Miranda problem, but the fact that the man did not know that his cellmate was working for the police.

47
Q

If I record an easement in a record that does not include the tract, does the next guy get constructive notice?

The recording officer maintains an alphabetical grantor-grantee index, but no tract index.

A

YES.

48
Q

Can judge call witnesses?

A Yes, but only if the witness has already been called and examined by one of the parties.

B Yes, but only if the witness is not testifying as an expert.

C Yes, but the parties are entitled to cross-examine the witness.

D No, because only parties may call and examine witnesses.

A

The judge may call and examine the witness, but the parties are entitled to cross-examine the witness. A court is entitled to examine any witness called by any party, and may also call a witness on its own or at a party’s request.

49
Q

During the trial of a personal injury case, the plaintiff calls a witness to testify that he saw the defendant spill a slippery substance in the roadway. Following the testimony of the witness, the defendant calls the witness’s neighbor, who testifies that the witness has a poor reputation for truthfulness in the community. The plaintiff’s attorney then cross-examines the neighbor, asking her, in good faith, if she committed the crime of false pretenses last year. Last year, the neighbor had in fact been charged with and convicted of the crime of false pretenses. The defendant’s attorney objects to this question.

Should the objection be sustained?

A No, because the neighbor was convicted of the crime of false pretenses.

B No, because the plaintiff’s attorney asked the question in good faith.

C Yes, because an impeaching witness cannot be impeached on collateral matters.

D Yes, because such an inquiry is not proper on cross-examination.

A

The question by the plaintiff’s attorney should be allowed because he was acting in good faith. A witness may be impeached by means of being interrogated upon cross-examination, in the discretion of the court, with respect to any act of misconduct that is probative of truthfulness (i.e., an act of deceit or lying). The cross-examiner must act in good faith with some reasonable basis for believing that the witness may have committed the bad act inquired about, but it is not required that the witness have been convicted of a crime. Here, the plaintiff’s attorney is attempting to cast an adverse reflection on the truthfulness of the neighbor. The commission of the crime of false pretenses involves the making of a false representation and is therefore an act of misconduct that is probative of the actor’s truthfulness. Thus, because the plaintiff’s attorney inquired as to this matter in good faith, his question is a permissible method of impeachment, and the objection of the defendant’s attorney should be overruled.

(A) is incorrect because it implies that the objection could be sustained if the neighbor was not convicted of the crime. As noted above, such an inquiry can be conducted regardless of whether the witness was convicted. Therefore, the objection to the plaintiff’s attorney’s good faith inquiry would be overruled even if the neighbor was not convicted of false pretenses.

50
Q

A state set up an intrastate message routing system to carry messages to and from the various state agency offices located throughout the state. This proved to be cheaper and more efficient than the United States Postal Service. The message service worked so well that the state offered the messenger service to its employees as a fringe benefit. Moreover, it expanded delivery options beyond state offices to any address in the state and permitted the employees to use the service for personal correspondence as well as for official business.

Are the state’s actions constitutional?

A Yes, because the messenger service operates entirely within the state borders.

B Yes, because the Commerce Clause does not prohibit states from acting as a market participant.

C No, because the Equal Protection Clause prohibits this singling out of state employees for special benefits.

D No, because it violates the federal postal monopoly.

A

The legislation is unconstitutional because it violates the federal postal monopoly. Article I, Section 8, Clause 7 of the Constitution grants Congress the power to establish post offices and post roads. This power grants Congress a monopoly over the delivery of mail. No other system for delivering mail—public or private—can be established absent Congress’s consent. Congress has delegated to the Postal Service the power to decide whether others may compete with it, and the Postal Service has carved out an exception to its monopoly for extremely urgent letters. However, this exception would not apply to the state messenger service here since the state service extends to every letter or package of an employee deliverable within the state.

(B) also is irrelevant. While it is true that there is a market participant exception to the Commerce Clause, the Commerce Clause is not the controlling law here; the controlling law is the federal postal power

51
Q

An attorney was employed by the United States Department of Health and Human Services in a regional office located in a tobacco-growing state. A labor contract between the agency and the clerical workers union contained a policy providing for termination of union employees only for certain specified grounds. The attorney, however, was not a member of the union and not covered by such a policy. The attorney was angered by the regional director’s refusal to adopt a no-smoking policy for employees and visitors in the office. She posted a notice in the employee cafeteria ridiculing what she called the hypocrisy of an agency promoting health issues and nonsmoking programs while refusing to provide its employees with those same opportunities. The notice prompted a great deal of debate among the employees and was brought to the attention of the regional director, who was very displeased.

Which of the following statements is most accurate regarding the director’s right to dismiss the attorney?

A The attorney has a liberty interest in the exercise of her First Amendment rights that entitles her to a hearing to contest the grounds of her dismissal.

B The attorney has a property interest as a public employee that precludes her from being fired without notice and an opportunity to respond.

C The attorney has no right to a hearing because her statements were not an expression of views on public issues.

D The attorney has both a liberty interest and a property interest that entitles her to a pre-termination evidentiary hearing.

A

If the attorney is fired, she has a right to a hearing to determine whether her First Amendment rights were violated by her dismissal.

Under the Due Process Clause of the Fifth Amendment, a person has a liberty interest in the exercise of specific rights provided by the Constitution, including freedom of speech.

If a government employer seeks to fire an employee for speech-related conduct when the speech involved a matter of public concern but is not made pursuant to her official duties, the courts must carefully balance the employee’s rights as a citizen to comment on a matter of public concern against the government’s interest as an employer in the efficient performance of public service. Under the Court’s expansive interpretation of what is a public issue in this context [see Rankin v. McPherson (1987)], the attorney’s statement would probably qualify. At the very least, she can make a sufficient showing that her termination violates her free speech rights to be entitled to a hearing on the issue under procedural due process principles. [See Givhan v. Western Line Consolidated School District (1979)]

(C) is wrong because the attorney is entitled to a hearing as long as she can raise a prima facie claim that her speech, which was regarding an important health issue and the perception of her agency, was on a public issue and therefore protected by the First Amendment.

(B) is wrong because the attorney does not appear to have a property interest in her job. A public employee who is subject to removal only for “cause” has a property interest in her job and generally must be given notice of the charges against her that are to be the basis for her job termination, and a pre-termination opportunity to respond to those charges. Here, however, the attorney did not have a property interest in her job; she could have been dismissed for no reason at all. She was not covered by the labor contract between the agency and its clerical workers, and there appears to be no other basis for her to claim an entitlement to continued employment

52
Q

Can you be convicted of larceny of stolen property?

A

(B) is incorrect because any personal property may be the subject of larceny, regardless of its origins.

53
Q

A young woman went to her local shoe shop and selected a pair of shoes. She gave the salesperson cash for the shoes. As the salesperson was putting the shoes into a bag, a robber brandishing a gun entered the store, forced the salesperson to put all of the money in the register into the bag with the shoes, and fled with the bag, the money, and the shoes. After the police had come, the young woman asked the salesperson to get her another pair of shoes. He told the young woman that she would have to pay for them again. The young woman refused.

If the young woman sues the shoe shop for another pair of shoes, who will prevail?

A The young woman, because she did not yet have possession of the shoes.

B The young woman, because the purpose of the contract had been made impossible by an unforeseen event.

C The shoe shop, because title to the shoes had already passed to the young woman.

D The shoe shop, because the contract goods had already been identified.

A

The young woman will prevail. Where the seller is a merchant, the risk of loss does not pass to the buyer until the buyer takes physical possession of the goods. (B) is wrong because performance was not impossible; the young woman had already performed and the shoe store could perform by tendering another pair of shoes. (C) is wrong because passage of title does not shift the risk of loss in this case. (D) is wrong because while a buyer gains some rights once the goods are identified, identification does not shift the risk of loss.

54
Q

After being notified by a doctor that her employment with his office was terminated, a nurse applied for a position with a hospital. In her application, the nurse listed her former employment with the doctor, with the understanding that the doctor might be contacted. The doctor, in response to a telephone inquiry from the hospital, stated that the nurse “lacked professional competence.” Although the doctor reasonably believed that to be a fair assessment of the nurse, his adverse rating was based on an episode of malpractice for which he blamed the nurse but which in fact was chargeable to another doctor. Because of the doctor’s adverse comment on her qualifications, the nurse was not employed by the hospital.

If the nurse asserts a claim based on defamation against the doctor, will the nurse prevail?

A Yes, because the doctor was mistaken in the facts on which he based his opinion of the nurse’s competence.

B Yes, because the doctor’s statement reflected adversely on the nurse’s professional competence.

C No, because the nurse authorized the hospital to make inquiry of her former employer.

D No, because the doctor had reasonable grounds for his belief that the nurse was not competent.

A

The nurse will not prevail because the doctor had reasonable grounds for his statement. As a former employer responding to queries of a prospective employer about a job applicant, the doctor has a qualified privilege. Such a privilege is not absolute; it exists only if exercised in a reasonable manner and for a proper purpose. The privilege may be lost if the speaker made a statement not within the scope of the privilege or if the speaker acted with actual malice (i.e., knowledge that the statement was untrue or with reckless disregard as to its truth or falsity). Because the doctor had reasonable grounds for his belief, he was not acting with actual malice.

(A) is incorrect because of the reasons stated in the analysis above. A statement of opinion may be actionable if it appears to be based on specific facts which, if expressly stated, would be defamatory. However, because of the qualified privilege, the doctor will not be liable for his mistake as long as his belief was reasonable.

(B) is incorrect because the fact that the statement was in a category that is slander per se (i.e., adversely reflecting on the nurse’s abilities to practice her profession) goes to whether the nurse must plead special damages. It does not, however, undermine the qualified privilege.

(C) is incorrect because permission to make inquiry is not tantamount to consent to be defamed.

55
Q

An underworld informer advised a police investigator that his neighbor was running an illegal bookmaking operation in his apartment, and that the informer had placed bets with the neighbor at this location. The officer obtained a search warrant, based on his affidavit reciting the foregoing facts, and further stating that the underworld informer was a person who had given him accurate information in previous cases, but whose identity could not be revealed because it might jeopardize other criminal investigations being carried on by the police. Armed with the search warrant, police officers went to the neighbor’s apartment. They entered when the neighbor opened the door and searched the apartment. They seized various wagering slips and bookmaking apparatus (described in the search warrant) and placed the neighbor under arrest for illegal gambling. Prior to trial, the neighbor challenges the validity of the search warrant.

Was the search warrant valid?

A No, because it was based on hearsay information.

B No, because the officer failed to disclose the identity of the informer, so that the accuracy of his information could not be verified.

C Yes, because the identity of the informer is never required.

D Yes, because the affidavit accompanying it is sufficiently detailed to allow a determination of probable cause.

A

The search warrant is valid because the affidavit accompanying it is sufficiently detailed to allow a determination of probable cause. A warrant must be based on a showing of probable cause. Along with a request for a warrant, a police officer must submit to a magistrate an affidavit setting forth sufficient underlying circumstances to enable the magistrate to make a determination of probable cause independent of the officer’s conclusions. The affidavit may be based on an informer’s statements. The sufficiency of the affidavit is evaluated according to the “totality of the circumstances.” There must be sufficient information for the magistrate to be able to make a common sense evaluation of probable cause. Among the factors determinative of probable cause are the informer’s reliability, credibility, and basis of knowledge. Here, the officer’s affidavit indicates that the informer has previously proved to be reliable by providing accurate information in other cases. This, in turn, enhances the credibility of the informer. Also, the informer’s knowledge is based on his having personally placed bets with the neighbor at his apartment. Thus, the officer’s affidavit is supported by sufficient underlying circumstances to allow a magistrate’s finding that there was a showing of probable cause.

(C) is incorrect because it is overbroad. An informer’s identity need not be revealed only if there is sufficient other evidence to make a probable cause determination.